Đến nội dung

minhtuyb nội dung

Có 497 mục bởi minhtuyb (Tìm giới hạn từ 30-03-2020)



Sắp theo                Sắp xếp  

#294700 Topic các bất đẳng thức lớp 8 hay dùng và các bài toán BĐT

Đã gửi bởi minhtuyb on 19-01-2012 - 17:18 trong Bất đẳng thức và cực trị

Đưa lên bài này cho mọi người tham khảo!
Bài 4: Cho $a,b,c>0;a+b+c=3$. CMR:
$\frac{a}{3a^2+5}+\frac{b}{3b^2+5}+\frac{c}{3c^2+5}\leq \frac{3}{8}$



#295019 Topic bất đẳng thức THCS (2)

Đã gửi bởi minhtuyb on 21-01-2012 - 13:43 trong Bất đẳng thức và cực trị

Bài 137: (không biết dễ hay khó :lol: Zz )
Quy nạp nhé ;)
Cho n số thực dương $x_1;x_2;...x_n$ có tích bằng 1. Chứng minh rằng
$x_1+x_2+...+x_n\geq n$

*Với $n=1$, BĐT đúng
*Giả sử BĐT đúng với $n=k$, tức là ta có:
$x_1+x_2+...+x_k\geq k(1)$ với $x_1.x_2.....x_k=1$ (giả thiết quy nạp)
-Cần c/m: $x_1+x_2+...+x_k+x_{k+1}\geq k$ với $x_1.x_2.....x_k.x_{k+1}=1$. Thật vậy, cộng 2 vế của (1) với $x_{k+1}$, ta có:
$x_1+x_2+...+x_k+x_{k+1}\geq k+x_{k+1}$
Mà $x_1.x_2.....x_k=1$ và $x_1.x_2.....x_k.x_{k+1}=1\rightarrow x_{k+1}=1$. Suy ra:
$x_1+x_2+...+x_k+x_{k+1}\geq k+x_{k+1}=k+1(Q.E.D)$
Vậy BĐT đúng với $n=k+1$. Theo giả thiết quy nạp thì BĐT đã cho đúng với $\forall n\in N*$



#296520 Tìm giá trị biểu thức: $P=\dfrac{1}{x}+\dfrac{1}{y}+\dfra...

Đã gửi bởi minhtuyb on 26-01-2012 - 11:45 trong Phương trình, hệ phương trình và bất phương trình

Cho x,y,z khác không thỏa mãn:
$\left\{\begin{matrix}
x(\dfrac{1}{y}+\dfrac{1}{z})+y(\dfrac{1}{x}+\dfrac{1}{z})z(\dfrac{1}{x}+\dfrac{1}{y})=-2 & \\
x^{2}+y^{2}+z^{2}=1 &
\end{matrix}\right.$
Tìm giá trị biểu thức: $P=\dfrac{1}{x}+\dfrac{1}{y}+\dfrac{1}{z}$

Tính đc đến đây thôi:
$x(\dfrac{1}{y}+\dfrac{1}{z})+y(\dfrac{1}{x}+\dfrac{1}{z})z(\dfrac{1}{x}+\dfrac{1}{y})=-2\Leftrightarrow \sum \frac{x}{y}+\frac{x}{z}=-2\Leftrightarrow \sum \frac{x}{x}+\frac{x}{y}+\frac{x}{z}=1$
$\Leftrightarrow \sum x(\frac{1}{x}+\frac{1}{y}+\frac{1}{z})=1\Leftrightarrow (x+y+z)(\frac{1}{x}+\frac{1}{y}+\frac{1}{z})=x^2+y^2+z^2(=1)
\Leftrightarrow \frac{1}{x}+\frac{1}{y}+\frac{1}{z}=\frac{x^2+y^2+z^2}{x+y+z}$



#297703 Chứng minh có đúng 1 trong ba số lớn hơn 1

Đã gửi bởi minhtuyb on 01-02-2012 - 17:52 trong Đại số

Bài 2: $\frac{a}{b}+\frac{b}{c}+\frac{c}{a}\geq 3\sqrt[3]{\frac{a}{b}.\frac{b}{c}.\frac{c}{a}}=3<Q.E.D>$
Dấu bằng khi $a=b=c$
Bài 3: $21(a+\frac{1}{b})+3(b+\frac{1}{a})=21a+3b+\frac{21}{b}+\frac{3}{a}=(\frac{3}{a}+\frac{a}{3})+(\frac{21}{b}+\frac{7b}{3})+\frac{62}{3}a+\frac{2}{3}b$
$\geq 2\sqrt{\frac{3}{a}.\frac{a}{3}}+2\sqrt{\frac{21}{b}.\frac{7b}{3}}+\frac{62}{3}.3+\frac{2}{3}.3=80<Q.E.D>$
Dấu bằng khi $a=b=3$

Bài 1: Cho ba số x,y,z khác 0 thỏa mãn:
$\left\{\begin{matrix} xyz=1\\ \frac{1}{x}+\frac{1}{y}+\frac{1}{z}< x+y+z \end{matrix}\right.$
Chứng minh rằng có đúng một trong ba số x,y,z lớn hơn 1.

$\left\{\begin{matrix} xyz=1\\ \frac{1}{x}+\frac{1}{y}+\frac{1}{z}< x+y+z \end{matrix}\right.\Leftrightarrow \left\{\begin{matrix} xyz=1\\ xyz(\frac{1}{x}+\frac{1}{y}+\frac{1}{z})< x+y+z \end{matrix}\right.$
$\Leftrightarrow \left\{\begin{matrix} xyz=1\\ xy+yz+zx< x+y+z \end{matrix}\right.\Leftrightarrow \left\{\begin{matrix} xyz=1\\ x+y+z-(xy+yz+zx)>0 \end{matrix}\right.$
Xét tích:
$(x-1)(y-1)(z-1)=xyz-(xy+yz+zx)+(x+y+z)-1=x+y+z-(xy+yz+zx)>0\Rightarrow (x-1)(y-1)(z-1)>0$
Vậy trong 3 số $x,y,z$ có 1 số lớn hơn 1, 2 số nhỏ hơn 1 hoặc cả 3 số lớn hơn 1
Tuy nhiên, nếu $x,y,z>1\Rightarrow xyz>1$. Mâu thuẫn với gt
Vậy ta có ĐPCM



#297795 Tìm B thuộc đường thẳng d:y=3 và C thuộc Ox sao cho tam giác ABC đều.

Đã gửi bởi minhtuyb on 02-02-2012 - 16:51 trong Phương trình - hệ phương trình - bất phương trình

Trên mp tọa độ cho A(1;1). tìm B thuộc đường thẳng d:y=3 và C thuộc Ox sao cho tam giác ABC đều.

Giúp em càng nhanh càng tốt nha, thanks nhiều :D



#297952 $2x^2-y^{14}=1$

Đã gửi bởi minhtuyb on 03-02-2012 - 21:00 trong Số học

Thử phát :):
Dễ thấy $y$ lẻ $\Rightarrow y^7$ lẻ. Đặt $y^7=2k+1 (k\in Z)$ thì:
$2x^2-y^14=1\Leftrightarrow 2x^2=y^14+1=(2k+1)^2+1 \Leftrightarrow 2x^2= 4k^2+4k+2\Leftrightarrow x^2=2k^+2k+1\Leftrightarrow x^2=(k+1)^2+k^2(1)$
-Với $k=0\Rightarrow x=\pm 1;y^7=1\Leftrightarrow x=\pm 1;y=1™$
-Với $k=-1\Rightarrow x=\pm 1;y^7=-1\Leftrightarrow x=\pm 1;y=-1™$
-Với $k\neq 0;-1$ thì (1) trở thành phương trình Py-ta-go. Lại có $k;k+1$ là hai số nguyên liên tiếp $\Rightarrow$ phương trình Py-ta-go có nghiệm duy nhất $x^2=5^2;(k+1)^2=4^2;k^2=3^2\Rightarrow x=\pm 5;k=\pm 3\Rightarrow x=\pm 5;y^7=\pm 3$. Pt vô nghiệm vì $y\not\in Z$
Vậy pt có nghiệm nguyên là :$(x;y)=(1;1);(1;-1);(-1;1);(-1;-1)$



#298026 Topic bất đẳng thức THCS (2)

Đã gửi bởi minhtuyb on 04-02-2012 - 14:30 trong Bất đẳng thức và cực trị

Bài 240: Cho x,y là các số thực thoả mãn $x^2+y^2=x+2$. Tìm GTNN và GTLN của biểu thức
$P=x+2y$

$P=x+2y\Rightarrow x=P-2y$.Theo gt ta có:
$(P-2y)^2+y^2=P-2y+2\Leftrightarrow P^2-4Py+4y^2+y^2-P+2y-2=0\Leftrightarrow 5y^2-2(2P-1)y+P^2-P-2=0$
Để pt bậc hai ẩn y có nghiệm thì :
$\Delta '=(2P-1)^2-5(P^2-P-2)\geq 0$
$\Leftrightarrow -P^2+P+11\geq 0$
Giải BPT trên thu đc $\frac{1-3\sqrt{5}}{2}\leq P\leq \frac{1+3\sqrt{5}}{2}$



#298080 Topic bất đẳng thức THCS (2)

Đã gửi bởi minhtuyb on 04-02-2012 - 21:58 trong Bất đẳng thức và cực trị

Bài 245: Cho các đa thức
$P(x)=x^3+ax^2+bx+c$, $Q(x)=x^2+x+2005$
Biết phương trình $P(x)=0$ có 3 nghiệm thực phân biệt còn phương trình P(Q(x))=0 vô nghiệm. CMR
$P(2005)>\frac{1}{64}$
Chuyên Thái Bình 2005-2006

-Gọi $x_1;x_2;x_3$ là 3 nghiệm thực phân biệt của pt $P(x)=0$. Theo định lý Bezout và do hệ số của $x^3$ bằng 1 $\Rightarrow P(x)=(x-x_1)(x-x_2)(x-x_3)$
-Suy ra:$P[Q(x)]=(Q(x)-x_1)(Q(x)-x_2)(Q(x)-x_3)$. do pt $P[Q(x)]=0$ vô nghiệm nên $Q(x)-x_1\neq 0;Q(x)-x_2\neq 0;Q(x)-x_3\neq 0$
-Với $Q(x)-x_1\neq 0\Rightarrow$ phương trình $x^2+x+2005-x_1=0$ vô nghiệm, suy ra:
$\Delta =1-4(2005-x_1)<0\Leftrightarrow 2005-x_1>\frac{1}{4}$
-Tương tự: $2005-x_2>\frac{1}{4};2005-x_3>\frac{1}{4}$
Vậy: $P(2005)=(2005-x_1)(2005-x_2)(2005-x_3)>(\frac{1}{4})=\frac{1}{64}<Q.E.D>$



#298256 Cực trị hình học và hình chiếu của một điểm trên 3 cạnh tam giác nội tiếp

Đã gửi bởi minhtuyb on 05-02-2012 - 22:24 trong Hình học

Có 2 bài xin thình giáo :D
Bài 1:Cho $\Delta ABC$ vuông tại A nội tiếp đường tròn tâm O đường kính BC. P là một điểm di chuyển trên nửa đường tròn không chứa A. Gọi $K,L,M$ lần lượt là hình chiếu của P trên $BC,AC,AB$. Đặt $PK=x;PL=y;PM=z;BC=a;AC=b;AB=c$. Tìm GTNN của:
$S=\frac{a}{x}+\frac{b}{y}+\frac{c}{z}$
Bài 2: Cho Cho $\Delta ABC$ nội tiếp đường tròn tâm O. Điểm M nằm trên cung nhỏ AC. Kẻ $ME\perp AC;MF\perp BC$. Gọi $P,Q$ lần lượt là trung điểm của $AB,EF$.
CMR:$\Delta PQM$ vuông



#298467 Cực trị hình học và hình chiếu của một điểm trên 3 cạnh tam giác nội tiếp

Đã gửi bởi minhtuyb on 07-02-2012 - 13:15 trong Hình học

Hóa ra hai bài này phải xài Sim-sơn :D.
Mình thắc mắc tí:

* $\frac{b}{y} +\frac{c}{z}=\frac{AC}{PL}+\frac{AB}{PM}=\frac{AL}{PL}+\frac{LC}{PL}+\frac{AM}{PM}-\frac{BM}{PM}$

Chỗ này chắc bạn hơi nhầm tí: $\frac{AC}{PL}+\frac{AB}{PM}=\frac{AL}{PL}+\frac{LC}{PL}+\frac{AM}{PM}+\frac{BM}{PM}$ (Nhầm dấu cộng thành trừ)

Dù sao cũng cảm ơn bạn nhiều :)

P/s:

Từ (1)(2)(3) $\frac{a}{x}=\frac{b}{y}+\frac{c}{z}\Rightarrow S=2.\frac{a}{x}$

Sao chỗ này mình thay góc vào không đc nhỉ. $\widehat{CPK}$ thay bằng góc gì đó



#298570 $4-m=\frac{2}{x+1}$

Đã gửi bởi minhtuyb on 08-02-2012 - 13:06 trong Phương trình, hệ phương trình và bất phương trình

ĐXKĐ: $x\neq -1$
$4-m=\frac{2}{x+1}\Leftrightarrow (4-m)(x+1)=2\Leftrightarrow (4-m)x+4-m=2\Leftrightarrow (4-m)x=m-2$ -Với $m=4$ pt vô nghiệm -Với $m\neq 4$ thì $x=\frac{m-2}{4-m}<0$ Lập bảng xét dấu đc $m>4$ hoặc $m<2$.



#298571 chứng minh rằng trong các tổng của 5 số theo mỗi cột , mỗi hàng hay mỗi đường...

Đã gửi bởi minhtuyb on 08-02-2012 - 13:17 trong Các dạng toán khác

-Người ta viết vào mỗi ô vuông một trong các số -1,1,hoặc 0 nên có thể xảy ra trường hợp các tổng của 5 số có thể là: $ -5;-4;-3;-2;-1;0;1;2;3;4;5$ (11 trường hợp)
-Tổng cộng có: 5 tổng hàng+5 tổng cột+2 tổng đường chéo=12 tổng. +Theo diricle có ĐPCM



#298641 $5\sqrt{1+x^{3}}=2x^{2}+4$

Đã gửi bởi minhtuyb on 08-02-2012 - 20:20 trong Phương trình, hệ phương trình và bất phương trình

B1: $5\sqrt{1+x^{3}}=2x^{2}+4$

ĐK:$x\geq -1$
$5\sqrt{1+x^{3}}=2x^{2}+4\Leftrightarrow 5\sqrt{(x+1)(x^2-x+1)}=2(x^2+2)(1)$
Đặt $a=\sqrt{x+1};b=\sqrt{x^2-x+1}(a;b\geq 0)\Rightarrow a^2+b^2=x+1+x^2-x+1=x^2+2$. Suy ra:
$(1)\Leftrightarrow 5ab=2(a^2+b^2)\Leftrightarrow 2a^2-5ab+b^2=0\Leftrightarrow (2a-b)(a-2b)=0\Leftrightarrow ...$



#298709 Topic bất đẳng thức THCS (2)

Đã gửi bởi minhtuyb on 09-02-2012 - 12:30 trong Bất đẳng thức và cực trị

Bài 261: Cho x,y,z $ \ge 0;x + y + z \le 3$
CMR: $$\frac{x}{{1 + x^2 }} + \frac{y}{{1 + y^2 }} + \frac{z}{{1 + z^2 }} \le \frac{3}{2} \le \frac{1}{{1 + x}} + \frac{1}{{1 + y}} + \frac{1}{{1 + z}}$$


$\sum \frac{x}{1+x^2}\leq \sum \frac{x}{2x}=\sum \frac{1}{2}=\frac{3}{2}(1)$
$\sum \frac{1}{1+x}\geq \frac{9}{1+x+1+y+1+y}\geq \frac{9}{3+3}=\frac{3}{2}(2)$
-Từ (1) và (2) có ĐPCM

Bài 267: Cho các số thực dương a,b,c thỏa mãn $3(ab+bc+ac)=1$
CMR: $$\frac{a}{{a^2 - bc + 1}} + \frac{b}{{b^2 - ac + 1}} + \frac{c}{{c^2 - ba + 1}} \ge \frac{1}{{a + b + c}}$$

$\sum \frac{a}{a^2-bc+1}=\sum \frac{a^2}{a^3-abc+a}\geq \frac{(a+b+c)^2}{a^3+b^3+c^3-3abc+a+b+c}$
$=\frac{(a+b+c)^2}{(a+b+c)(a^2+b^2+c^2-ab-bc-ca)+a+b+c}=\frac{a+b+c}{a^2+b^2+c^2-ab-bc-ca+1}$
$=\frac{a+b+c}{a^2+b^2+c^2-ab-bc-ca+3(ab+bc+ca)}=\frac{a+b+c}{(a+b+c)^2}=\frac{1}{a+b+c}<Q.E.D>$

Bài 262: Cho a,b,c là 3 số thực dương thay đổi thỏa mãn a+b+c=3
CMR: $$A = \frac{{a^3 }}{{b(2c + a)}} + \frac{{b^3 }}{{c(2a + b)}} + \frac{{c^3 }}{{a(2b + c)}} \ge 1$$

$\sum \frac{a^3 }{b(2c + a)}=\sum \frac{a^3}{ab+2bc}$
Có: $\frac{a^3}{ab+2bc}+\frac{ab+2bc}{9}+\frac{1}{3}\geq 3\sqrt[3]{\frac{a^3}{ab+2bc}.\frac{ab+2bc}{9}.\frac{1}{3}}=a$
$\Rightarrow \sum\frac{a^3}{ab+2bc}+\sum\frac{ab+2bc}{9}+\sum\frac{1}{3}\geq \sum a\Rightarrow \sum\frac{a^3}{ab+2bc}+\frac{ab+bc+ca}{3}+1\geq 3$
$\Rightarrow \sum\frac{a^3}{ab+2bc}\geq 2-\frac{ab+bc+ca}{3}\geq 2-\frac{(a+b+c)^2}{3.3}=1<Q.E.D>$
Dấu bằng khi $a=b=c=1$



#298734 Topic bất đẳng thức THCS (2)

Đã gửi bởi minhtuyb on 09-02-2012 - 17:30 trong Bất đẳng thức và cực trị

Nếu trùng xóa hộ :D:
Bài 269: Cho $|x|\leq 1;n\in N*$, CMR:
$(1+x)^n+(1-x)^n \leq 2^n$



#298739 CMR:$cosA+cosB+ cos C \leq \frac{3}{2}$

Đã gửi bởi minhtuyb on 09-02-2012 - 18:02 trong Hình học

Bài này xuất hiện nhiều rồi, bạn search là thấy:

File gửi kèm  VNMATH.COM-12cachchung minh bdt.pdf   69.26K   78 Số lần tải



#299473 CMR: $AC^2+BC^2=4R^2$

Đã gửi bởi minhtuyb on 15-02-2012 - 11:55 trong Hình học

Phân giác trong và ngoài của góc $widehat{ACB}$ cắt AB tại L,M. Biết $CL=CM$, R là bán kính đường tròn ngoại tiếp $\Delta ABC$ CMR: $AB^2+AC^2=4R^2$



#299529 Bài 1: $\sqrt{5-x}+\sqrt{x-1}=-x^{2}+2x+1$

Đã gửi bởi minhtuyb on 15-02-2012 - 20:07 trong Phương trình - hệ phương trình - bất phương trình

Bài 1: $1\leq x\leq 5$
$VT^2=(\sqrt{5-x}+\sqrt{x-1})^2=4+2\sqrt{(5-x)(x-1)}\geq 4\Rightarrow VT\geq 2$
Dấu bằng khi $x=1$ hoặc $x=5$
$VP=-(x^2-2x+1)+2=-(x-1)^2+2\leq 2$
Dấu bằng khi $x=1$
-Kết hợp ta thấy pt có no duy nhất $x=1$



#300038 Topic bất đẳng thức THCS (2)

Đã gửi bởi minhtuyb on 19-02-2012 - 19:17 trong Bất đẳng thức và cực trị

Post thêm vài bài duy trì pic đã, trầm quá :wacko:
Bài 271: Cho $x,y,z>0$ thỏa mãn $xy^2z^2+x^2z+y=3z^2$. Tìm max của biểu thức:
$P=\frac{z^4}{1+z^4(x^4+y^4)}$
Bài 272: Cho $a,b,c>1$, tìm min của biểu thức:
$Q=\frac{a}{\sqrt{b}-1}+\frac{b}{\sqrt{c}-1}+\frac{c}{\sqrt{a}-1}$
Bài 273: Cho $a,b,c$ là những số thực dương thỏa mãn $a+b+c=3$, tìm min của:
$R=\sum \frac{a^2}{\sqrt{3a^2+8b^2+14ab}}$

Chú ý:Các anh chị THPT để cho bon em THCS làm cái, bị tranh hết mất :icon6:



#300066 Trận 1- "MSS01 SubjectMath" VS ALL

Đã gửi bởi minhtuyb on 19-02-2012 - 21:44 trong Thi giải toán Marathon cấp THCS 2012

Bài làm của minhtuyb:
-Từ $(36x+y)(36y+x)=2^z(1);x,y\geq 1\Rightarrow 2^z\geq 37^2\Rightarrow 36x+y;36y+z\vdots 2\Rightarrow x,y\vdots 2\Rightarrow x+y\vdots 2$
-Đặt $36x+y=2^a;36y+x=2^b(a,b\in N*;a+b=z)$. Không mất tính tổng quát, giả sử $a\geq b\Rightarrow a=b+k(k\in N)$. Ta có:
$\left\{\begin{matrix}36x+y=2^a=2^{b+k}\\ 36y+x=2^b\end{matrix}\right.$
Cộng vế với với của 2 phương trình trên, ta có:
$36x+y+36y+x=2^{b+k}+2^b\Leftrightarrow 37(x+y)=2^b(2^k+1)(2)$
TH1:Với $k=0\Leftrightarrow a=b\Leftrightarrow 36x+y=36y+x\Leftrightarrow x=y$, thay $x=y$ vào (1) ta có:
$37x.37x=2^z$. PT vô nghiệm nguyên dương vì $37\not\vdots 2$
TH2:Với $k>0\Rightarrow 2^k+1$ lẻ, kết hợp với $ x+y\vdots 2$ nên từ (2) ta có cách phân tích duy nhất:
$\left\{\begin{matrix}37=2^k+1\\ x+y=2^b\end{matrix}\right.\Leftrightarrow \left\{\begin{matrix}2^k=36\\ x+y=2^b\end{matrix}\right.$ Hệ pt vô nghiệm nguyên dương do pt $2^k=36$ vô nghiệm nguyên dương $\Rightarrow $ phương trình (1) vô nghiệm nguyên dương

Vậy phương trình đã cho không có nghiệm nguyên dương

Kết quả:
D-B=21.7
E=10
F=1 * 10=10
S=66.3



#300085 Trận 1- "MSS01 SubjectMath" VS ALL

Đã gửi bởi minhtuyb on 19-02-2012 - 22:40 trong Thi giải toán Marathon cấp THCS 2012

Bài toán mở rộng của minhtuyb:
Tìm nguyện nguyên dương của phương trình:
$[2mx+(2n+1)y][(2m+1)x+2ny]=2^z$ (với $m,n$ là tham số, $m,n\in N*$)

Giải

Bài làm của minhtuyb:
-Từ $[2mx+(2n+1)y][(2n+1)x+2may]=2^z(1);m,n,z,a,b\geq 1\Rightarrow 2mx+(2n+1)y;(2n+1)x+2my\vdots 2\Rightarrow x,y\vdots 2\Rightarrow x+y\vdots 2$
-Đặt $2mx+(2n+1)y=2^a;(2n+1)x+2my=2^b(3)(a,b\in N*;a+b=z)$. Không mất tính tổng quát, giả sử $a\geq b\Rightarrow a=b+k(k\in N)$. Ta có:
$\left\{\begin{matrix}2mx+(2n+1)y=2^a=2^{b+k}\\(2n+1)x+2my=2^b\end{matrix}\right.$
Cộng vế với với của 2 phương trình trên, ta có:
$2mx+(2n+1)y+(2n+1)x+2my=2^{b+k}+2^b \Leftrightarrow (2m+2n+1)(x+y)=2^b(2^k+1)(2)$
TH1:Với $k=0\Leftrightarrow a=b\Leftrightarrow 2mx+(2n+1)y=(2n+1)x+2my\Leftrightarrow (x-y)(2m-2n-1)=0(3)$. Vì $2m-2n-1$ lẻ nên không xảy ra $2m-2n-1=0$ nên ta có $(3)\Leftrightarrow x=y$ thay $x=y$ và $a=b$ vào (1) ta có:
$(4m+1)^2.x^2=2^z$. PT vô nghiệm nguyên dương vì $4a+1\not\vdots 2$
TH2:Với $k>0\Rightarrow 2^k+1$ lẻ, kết hợp với $ x+y\vdots 2$ nên từ (2) ta có cách phân tích duy nhất:
$\left\{\begin{matrix}2m+2n+1=2^k+1\\ x+y=2^b\end{matrix}\right.\Leftrightarrow \left\{\begin{matrix}m+n=2^{k-1}\\ x+y=2^b\end{matrix}\right.$
-Thay $2^b=x+y$ vào (3), ta có:
$(2n+1)x+2my=x+y\Leftrightarrow 2nx+2my=y$. Dễ thấy pt vô nghiệm vì $VT>VP$ với $\forall n,m,x,y\in N*$
Vậy pt tổng quát không có nghiệm nguyên



#300146 Biết $|a+b+c| \leq 1$ và $|\frac{a}{4} + \frac{...

Đã gửi bởi minhtuyb on 20-02-2012 - 17:03 trong Bất đẳng thức và cực trị

Tham khảo tại:
http://diendan.hocma...217&postcount=8



#300172 Topic bất đẳng thức THCS (2)

Đã gửi bởi minhtuyb on 20-02-2012 - 19:29 trong Bất đẳng thức và cực trị

Bài 275. Cho các số thực dương thoả mãn :$x + y + z + 1 = 4xyz$. Chứng minh rằng :
$$xy + yz + zx \ge x + y + z$$
Bài 276. Cho các số thực dương thoả mãn $a + b + c = 1$. Tìm GTLN của :
$$P = \sqrt{\dfrac{ab}{c + ab}} + \sqrt{\dfrac{bc}{a + bc}} + \sqrt{\dfrac{ca}{b + ca}} $$

Bài 275:Em chịu chỉ còn cách tra gg, tra ra thì lại thấy xài cả schur, pó tay :ukliam2:
Biến đổi giả thiết thành $\frac{1}{2x+1}$+$\frac{1}{2y+1}$+$\frac{1}{2z+1}$=1
Đặt a=$\frac{1}{2x+1}$; b=$\frac{1}{2y+1}$; c=$\frac{1}{2z+1}$ thì a,b,c >0 và a+b+c=1
=> x=$\frac{b+c}{2a}$; y=$\frac{c+a}{2b}$; z=$\frac{a+b}{2c}$
Do đó VT = $\frac{{a}^{2}+{b}^{2}+{c}^{2}}{abc}$+9
VP = $\frac{2(ab+bc+ca)}{abc}$
đpcm <=> ${a}^{2}+{b}^{2}+{c}^{2}+9abc \geq 2(ab+bc+ca)$
<=> (${a}^{2}+{b}^{2}+{c}^{2})(a+b+c) +9abc \geq 2(ab+bc+ca)(a+b+c)$
<=> $a(a-b)(a-c) + b(b-c)(b-a) + c(c-a)(c-b) \geq 0$
Đây là BĐT Schur
Nguồn:MS

Bài 276:
Từ gt: $a+b+c=1\Rightarrow c+ab=c(a+b+c)+ab=c^2+ac+bc+ab=(c+a)(c+b)$
Vậy ta có:
$P=\sum \sqrt{\frac{ab}{c+ab}}=\sum \sqrt{\frac{ab}{(c+a)(c+b)}}\leq ^{Cauchy nguoc} \sum (\frac{a}{c+a}+\frac{b}{c+b})=3$
Dấu bằng xảy ra khi $a=b=c=\frac{1}{3}$
Vậy $maxP=3$ khi $a=b=c=\frac{1}{3}$



#300306 Cho các số a,b,c dương CM các bất đẳng thức $\frac{a^{3}}{b}+...

Đã gửi bởi minhtuyb on 21-02-2012 - 13:43 trong Bất đẳng thức và cực trị

@@1: Theo Cauchy:
$\frac{a^3}{b}+ab+a^2\geq 3\sqrt[3]{\frac{a^3}{b}.ab.a^2}=3a^2$
Xây dựng các BĐT tương tự rồi cộng vào ta có:
$\sum \frac{a^3}{b}+\sum ab+\sum a^2\geq 3\sum a^2\Leftrightarrow \sum \frac{a^3}{b}\geq 2\sum a^2-\sum ab\geq \sum ab(Q.E.D)$
Dấu bằng khi $a=b=c$
@@2: $\sum \frac{1}{a^3+b^3+1}\leq 1$ mới đúng chứ nhỉ?



#300308 Tìm $x$ để $B$ = $0$

Đã gửi bởi minhtuyb on 21-02-2012 - 14:03 trong Số học

Bài này..... :wacko: ;
$$B=(x+3)(x^2+4x+4)=0\Leftrightarrow (x+3)(x+2)^2=0$$
Vậy với $x=-2;-3$ thì B bằng không
P/s:Đơn giản là phương trình tích thôi :wub: